- Mon Nov 06, 2017 4:58 pm
#41256
Complete Question Explanation
(The complete setup for this game can be found here: lsat/viewtopic.php?t=6505)
The correct answer choice is (E)
If Zone 1 has more sales representatives than Zone 3, only one Numerical Distribution is possible: 4-1-2. And, the only template under which a 4-1-2 distribution can occur is Template #2. The inclination at this point may be to draw out the entire solution to Template #2 in a 4-1-2 arrangement, but there is already enough information from the template and the distribution to eliminate the four incorrect answers without wasting the time to draw out the full solution:
Answer choice (A) is incorrect because in a 4-1-2 distribution only T can work in Zone 2.
Answer choice (B) is incorrect because in a 4-1-2 distribution only T can work in Zone 2.
Answer choice (C) is incorrect because in Template #2, P must work in Zone 1.
Answer choice (D) is incorrect because in Template #2, T must work in Zone 2.
Thus, by process of elimination answer choice (E) is proven correct.
(The complete setup for this game can be found here: lsat/viewtopic.php?t=6505)
The correct answer choice is (E)
If Zone 1 has more sales representatives than Zone 3, only one Numerical Distribution is possible: 4-1-2. And, the only template under which a 4-1-2 distribution can occur is Template #2. The inclination at this point may be to draw out the entire solution to Template #2 in a 4-1-2 arrangement, but there is already enough information from the template and the distribution to eliminate the four incorrect answers without wasting the time to draw out the full solution:
Answer choice (A) is incorrect because in a 4-1-2 distribution only T can work in Zone 2.
Answer choice (B) is incorrect because in a 4-1-2 distribution only T can work in Zone 2.
Answer choice (C) is incorrect because in Template #2, P must work in Zone 1.
Answer choice (D) is incorrect because in Template #2, T must work in Zone 2.
Thus, by process of elimination answer choice (E) is proven correct.